Cardio HW 2a-i Flashcards

1
Q

A “runaway” pacemaker is characterized by: A. an absence of pacemaker spikes.
B. profound slowing of the heart rate.
C. a tachycardic pacemaker rhythm.
D. a narrowing of the QRS complexes.

A

C

How well did you know this?
1
Not at all
2
3
4
5
Perfectly
2
Q

A 17-year-old man complains of palpitations and lightheadedness that began suddenly about 20 minutes ago. His blood pressure is 118/74 mm Hg, heart rate is rapid and regular, and respirations are 18 breaths/min. The cardiac monitor reveals a narrow QRS complex tachycardia at 180 beats/min. As you are applying supplemental oxygen, the cardiac rhythm spontaneously converts to a sinus rhythm. Closer evaluation of his rhythm reveals a rapid upslope to the R wave immediately after the end of the P wave. Which of the following statements regarding this scenario is correct?
Choose one answer.
A. This patient likely has Wolff-Parkinson-White syndrome.
B. The patient’s ECG abnormality is called an Osborn wave.
C. The ECG abnormality is caused by failure of the AV node.
D. In this patient, there is a delay in ventricular depolarization.

A

A

How well did you know this?
1
Not at all
2
3
4
5
Perfectly
3
Q

A 33-year-old woman presents with an acute onset of “fluttering” in her chest. She is conscious and alert but is somewhat anxious. She denies any significant medical problems but states that she has been under a lot of stress at work. You apply the cardiac monitor, which reveals a narrow QRS complex tachycardia at a rate of 170 beats/min. The patient’s blood pressure is 140/90 mm Hg, and she is breathing without difficulty. The MOST appropriate treatment for this patient involves:
Choose one answer.
A. oxygen, vagal maneuvers, and emotional support.
B. vagal maneuvers, IV access, and 0.25 mg/kg of diltiazem.
C. oxygen, emotional support, and 2.5 mg of midazolam IM.
D. oxygen, IV access, vagal maneuvers, and 6 mg of adenosine.

A

D

How well did you know this?
1
Not at all
2
3
4
5
Perfectly
4
Q

A 39-year-old man in asystole has been unresponsive to high-quality CPR and two doses of epinephrine. The patient is intubated and an IO catheter is in place. You should focus on:
Choose one answer.
A. establishing a peripheral IV line
B. providing mild hyperventilation.
C. searching for reversible causes.
D. transcutaneous cardiac pacing

A

C

How well did you know this?
1
Not at all
2
3
4
5
Perfectly
5
Q
A 41-year-old man complains of chest heaviness and mild shortness of breath that began about 2 hours ago. He is conscious and alert. As you are assessing him, he tells you that he has high blood pressure for which he takes Clonidine. His blood pressure is 160/90 mm Hg, heart rate is 140 beats/min and regular, and respirations are 22 breaths/min and somewhat labored. The cardiac monitor displays a narrow complex tachycardia in lead II. Which of the following interventions is NOT indicated for this patient?
Choose one answer.
	 A. Aspirin  	
	 B. Adenosine  	
	 C. IV access  	
	 D. 12-Lead ECG
A

B

How well did you know this?
1
Not at all
2
3
4
5
Perfectly
6
Q

A 55-year-old man complains of severe pain between his shoulder blades, which he describes as “ripping” in nature. He tells you that the pain began suddenly and has been intense and unrelenting since its onset. His medical history includes hypertension, and he admits to being noncompliant with his antihypertensive medication. Which of the following assessment findings would MOST likely reinforce your suspicion regarding the cause of his pain?
Choose one answer.
A. Disappearance of radial pulses during inspiration
B. Difference in blood pressure between the two arms
C. ST-segment depression on the 12-lead ECG tracing
D. Bruits to both carotid arteries during auscultation

A

B

How well did you know this?
1
Not at all
2
3
4
5
Perfectly
7
Q

A 56-year-old man complains of chest tightness, shortness of breath, and nausea. During your assessment, you note that he appears confused. He is profusely diaphoretic and has a blood pressure of 98/68 mm Hg and a rapid radial pulse. The cardiac monitor reveals a wide QRS complex tachycardia at a rate of 200 beats/min. After administering high-flow oxygen, you should:
Choose one answer.
A. attempt to slow his heart rate with vagal maneuvers and then start an IV line.
B. establish IV access, consider sedation, and perform synchronized cardioversion.
C. establish vascular access and administer 150 mg of amiodarone over 10 minutes.
D. obtain a 12-lead ECG tracing to determine the origin of his tachycardic rhythm.

A

B

How well did you know this?
1
Not at all
2
3
4
5
Perfectly
8
Q
A 56-year-old man presents with an acute onset of chest pressure and diaphoresis. He has a history of hypertension and type 2 diabetes. His airway is patent and his breathing is adequate. You should:
Choose one answer.
	 A. establish vascular access.  	
	 B. obtain baseline vital signs.  	
	 C. administer supplemental oxygen.  	
	 D. acquire a 12-lead ECG tracing.
A

C

How well did you know this?
1
Not at all
2
3
4
5
Perfectly
9
Q

A 67-year-old man presents with severe dyspnea, coarse crackles to all lung fields, and anxiety. He has a history of several myocardial infarctions and hypertension. Which of the following interventions will have the MOST immediate and positive effect?
Choose one answer.
A. Positive end-expiratory pressure ventilation
B. IV or IO access and 20 to 40 mg of furosemide
C. 0.4 mg sublingual nitroglycerin, up to three doses
D. Supplemental oxygen via nonrebreathing mask

A

A

How well did you know this?
1
Not at all
2
3
4
5
Perfectly
10
Q

A 68-year-old woman presents with an acute onset of confusion, shortness of breath, and diaphoresis. Her blood pressure is 72/50 mm Hg, her heart rate is slow and weak, and her respirations are increased and shallow. The ECG reveals a third-degree heart block at a rate of 38 beats/min. After placing the patient on high-flow oxygen, you should:
Choose one answer.
A. start an IV and administer 0.5 mg atropine.
B. obtain a 12-lead ECG to detect an acute myocardial infarction.
C. obtain vascular access and give a fluid bolus.
D. immediately attempt transcutaneous pacing.

A

D

How well did you know this?
1
Not at all
2
3
4
5
Perfectly
11
Q

A 70-year-old man called 9-1-1 because of generalized weakness. When you arrive at the scene, you find the patient seated in his recliner. He is conscious and alert and is breathing without difficulty. Your physical exam reveals tenderness to his right upper abdominal quadrant, edema to his ankles, and distended jugular veins. The patient tells you that he takes Vasotec for hypertension and Maxide for his swollen ankles. His vital signs are stable. The MOST appropriate treatment for this patient includes:
Choose one answer.
A. an IV of D5W, 0.4 mg of sublingual nitroglycerin, ECG, and transport.
B. high-flow oxygen, vascular access, 1 mg/kg of furosemide, and transport.
C. oxygen, cardiac monitoring, an IV line at a keep-open rate, and transport.
D. 12-lead ECG acquisition, vascular access, 4 mg of morphine, and transport.

A

C

How well did you know this?
1
Not at all
2
3
4
5
Perfectly
12
Q

A 70-year-old woman remains in asystole following 10 minutes of well-coordinated CPR, successful intubation, IV therapy, and three doses of epinephrine. There are no obvious underlying causes that would explain her cardiac arrest. At this point, it would be appropriate to:
Choose one answer.
A. attempt transcutaneous cardiac pacing.
B. defibrillate one time in case she is in V-Fib.
C. seriously consider ceasing resuscitative efforts.
D. transport at once with CPR continuing en route.

A

C

How well did you know this?
1
Not at all
2
3
4
5
Perfectly
13
Q

During the refractory period:
Choose one answer.
A. the heart is in a state of partial repolarization.
B. the heart is partially charged, but cannot contract.
C. the cell is depolarized or in the process of repolarizing.
D. the heart muscle is depleted of energy and needs to recharge.

A

C

How well did you know this?
1
Not at all
2
3
4
5
Perfectly
14
Q

Electrical capture during transcutaneous cardiac pacing is characterized by:
Choose one answer.
A. the presence of a strong pulse, despite a slow rate.
B. a pacemaker spike followed by a wide QRS complex.
C. narrow QRS complexes that are preceded by a pacemaker spike.
D. low-amplitude QRS complexes preceded by a pacemaker spike.

A

B

How well did you know this?
1
Not at all
2
3
4
5
Perfectly
15
Q

Epinephrine is used to treat patients in anaphylactic shock because of its effects of:
Choose one answer.
A. vasodilation and bronchoconstriction.
B. bronchodilation and vasoconstriction.
C. increased heart rate and automaticity.
D. parasympathetic nervous system blockade.

A

B

How well did you know this?
1
Not at all
2
3
4
5
Perfectly
16
Q

Fibrinolysis may be contraindicated in all of the following, EXCEPT:
Choose one answer.
A. major trauma or surgery within the past 4 weeks.
B. a history of structural central nervous system disease.
C. a history of anaphylactic shock caused by salicylates.
D. significant closed head trauma within the past 3 weeks.

A

C

17
Q

Fibrinolytic medications are beneficial to certain patients with an acute myocardial infarction because they:
Choose one answer.
A. decrease circulating platelets and thin the blood.
B. convert plasminogen to plasmin and destroy a clot.
C. destroy a clot by releasing fibrin into the bloodstream.
D. break down the plasmin concentration inside a blood clot.

A

B

18
Q

If the heart’s secondary pacemaker becomes ischemic and fails to initiate an electrical impulse:
Choose one answer.
A. the AV junction will begin pacing at 40 to 60 times/min.
B. you will see a brief period of bradycardia followed by asystole.
C. the P wave and PR interval will have an abnormal appearance.
D. you should expect to see a heart rate slower than 40 beats/min.

A

D

19
Q
In contrast to the pain associated with an acute myocardial infarction, pain from a dissecting aortic aneurysm:
Choose one answer.
	 A. often waxes and wanes.  	
	 B. gradually becomes severe.  	
	 C. is maximal from the onset.  	
	 D. is preceded by other symptoms.
A

C

20
Q

In contrast to the right side of the heart, the left side of the heart:
Choose one answer.
A. drives blood out of the heart against the relatively high resistance of the systemic circulation.
B. is a high-pressure pump that sends blood through the pulmonary circulation and to the lungs.
C. is a relatively low-pressure pump that must stretch its walls in order to force blood through the aorta.
D. drives blood out of the heart against the relatively low resistance of the pulmonary circulation.

A

A

21
Q

In contrast to treatment for supraventricular tachycardia, treatment for multifocal atrial tachycardia in the prehospital setting:
Choose one answer.
A. is often more effective.
B. involves atropine sulfate.
C. is generally not effective.
D. includes synchronized cardioversion.

A

C

22
Q
In the context of cardiac compromise, syncope occurs due to:
Choose one answer.
	 A. an increase in vagal tone.  	
	 B. a drop in cerebral perfusion.  	
	 C. a sudden cardiac dysrhythmia.  	
	 D. an acute increase in heart rate.
A

B

23
Q
In which of the following situations would you likely NOT be able to palpate a pulse despite effective chest compressions?
Choose one answer.
	 A. Profound hypoxia  	
	 B. Severe acidosis  	
	 C. Hyperkalemia  	
	 D. Tension pneumothorax
A

D

24
Q

Infarctions of the inferior myocardial wall are MOST often caused by:
Choose one answer.
A. blockage of the left coronary artery.
B. acute spasm of the circumflex artery.
C. occlusion of the right coronary artery.
D. a blocked left anterior descending artery.

A

C

25
Q
Injury to or disease of the \_\_\_\_\_\_\_\_\_\_\_\_\_\_ may cause prolapse of a cardiac valve leaflet, allowing blood to regurgitate from the ventricle into the atrium.
Choose one answer.
	 A. coronary sulcus  	
	 B. chordae tendineae  	
	 C. interatrial septum  	
	 D. coronary sinus
A

B